An object horizontally dragged across the surface by a 100 N force acting parallel to the surface . Find out the amount of work done by force in moving the object through a distance of 8 m

Answers

Answer 1

Answer:

800J

Explanation:

Given data

Force= 100N

Distance moved= 8m

The expression for the work done is

WD= Force* distance

substitute

WD= 100*8

WD= 800Joules

Hence the work done is 800J


Related Questions

How does the fur of the polar bear help it to live in the arctic region?

Answers

Answer:

It has 2 uses

1. Polar bears have white fur so that they can camouflage into their environment. Their coat is so well camouflaged in Arctic environments that it can sometimes pass as a snow drift.

2. They have a thick layer of body fat, which keeps them warm while swimming, and a double-layered coat that insulates them from the cold Arctic air.

When driving on a highway, sudden strong cross wind gusts: Select one: a. Do not affect a car as much as a strong head wind b. Affect large cars more than small cars c. Always cause severe dust problems d. Can move a car sideways into another lane

Answers

When driving on a highway, sudden strong cross wind gusts move a car sideways into another lane (option D).

What is a crosswind?

A crosswind is a wind blowing across a line of travel.

Accidents have been reported to be caused by strong winds and can cause heavy distractions that will cause the driver to lose control.

However, a strong and sudden crosswind will cause the car to move sideways into another lane, and can lead to severe accident.

Learn more about crosswind at: https://brainly.com/question/2814024

#SPJ1

A person performs a bench press. The bar's mass is 40 kg. When this person's arms are extended, the bar is 0.6 m above the starting point. How much work did the person do to push the bar off his chest to full extension of his arms

Answers

The work done by the person is 235.2 J.

Work done by the person

The work done by the person is calculated as follows;

W = mgh

where;

m is mass of the barheight through which the bar is raised

W = 40 x 9.8 x 0.6

W = 235.2 J

Thus, the work done by the person is 235.2 J.

Learn more about work done here: https://brainly.com/question/8119756

#SPJ1

The decomposition of H2O2 is a first-order reaction given by the formula: Rate = k [H2O2]1. How would the reaction rate be affected if the H2O2 concentration were double its original value?

Answers

Answer: The decomposition of H₂O₂ is a first-order reaction. Then, the reaction rate will be 2 times the original rate, if we double its concentration value.

Explanation: To find the answer, we need to know about the decomposition of H₂O₂.

What is the rate of reaction, if we double the concentration of H₂O₂?We have the equation of decomposition of H₂O₂ as,

                              2H₂O₂→ 2 H₂O+ O₂

We have given that, the equation of reaction rate of the first order reaction as,

                              Rate = k [H₂O₂]1

As we know that the equation of reaction rate is,

              Rate = rate constant × concentration of [H₂O₂]

Thus, by comparing both the equations, we get,

         rate constant=k and the concentration of [H₂O₂] = 1[H₂O₂].

Given that the concentration is doubled. Then the expression for rate will be,

                              Rate = 2k [H₂O₂].

Thus, we can conclude that the rate of reaction will be 2 times the initial rate if we double the concentration.

Learn more about the decomposition of H₂O₂ here:

https://brainly.com/question/28047273

#SPJ1

If we double the concentration value, the reaction rate will be twice as fast as before.

In order to determine the solution, we must understand how H2O2 breaks down.

If we double the concentration of H2O2, what will happen to the reaction rate?The formula for the breakdown of H2O2 is,

                        2H₂O₂→ 2 H₂O+ O₂

We have provided the first order reaction's reaction rate equation as,

                      Rate = [H2O2] k

As we are aware, the reaction rate equation is

                   Rate = rate constant × [H2O2] concentration

As a result, by comparing the two equations, we obtain, rate constant is k, and the [H2O2] concentration is 1 [H2O2].The concentration has doubled as a result. Afterward, the rate expression will be,

                           Rate = [H2O2] 2k.

As a result, we can infer that doubling the concentration will cause the reaction rate to double.

Learn more about how H2O2 breaks down here:

https://brainly.com/question/28047273

#SPJ1

A parallel plate capacitor fully charged to voltage V is connected to the battery (the voltage on the plates remains fixed). If the plates are pulled away from each other, how would it affect the charge on the capacitor

Answers

Charge will decreases.

A parallel plate capacitor when it is fully charged to voltage V is given as:

                    C = Q/V

The capacitance of parallel plate capacitor with two plates of Area A separated by distance d and no dielectric material between plates is

                    C = ε₀ A /d

since from above equation it shows C is proportional to Q and also C is inversely proportional to distance d.

So, ATQ when d increases C will decrease which in result decreases charge on the capacitor.

Thus,  Charge will decrease.

Learn more about capacitance here:

     https://brainly.com/question/17115454

          #SPJ4

Cindy's eye exam reveals that the reason she cannot get a sharp focus is that she has ___, an uneven curvature of the cornea or lens that prevents a sharp focus of an image on her retina.

Answers

Answer: Cindy's eye exam reveals that the reason she cannot get a sharp focus is that she has Astigmatism, an uneven curvature of the cornea or lens that prevents a sharp focus of an image on her retina.

Explanation: To find answer, we need to know more about the Astigmatism.

What is astigmatism?It is the defect of vision in which a person cannot simultaneously see both the vertical and horizontal views of an object with the same clarity.It is due to the irregular curvature of cornea, or the lens prevents a sharp focus of an image on retina.It can be corrected using a cylindrical lens.

Thus, from the above explanation, we can conclude that, the reason she cannot get a sharp vision is due to Astigmatism.

Learn more about Astigmatism here:

https://brainly.com/question/28044813

#SPJ4

A population distribution shows _______. a. the distribution of age groups within a population b. the amount of organisms in a unit area c. the total number of organisms in a population d. where and in what arrangement organisms are located Please select the best answer from the choices provided A B C D

Answers

The answer would be D where and in what arrangement organisms are located.

Frank and Lisa are analyzing the chart, which shows the speed at which light travels through different media.

Answers

Frank is correct because light travels through diamond at  the slowest speed, and speed and frequency have an inverse  relationship .

Lisa is correct because  light travels through air at the fastest speed of ,and speed and frequency have a direct relationship .

Neither  Frank nor Lisa  is correct because light  does not change frequency when it travels through different media .

Both Frank and Lisa are correct because light has both inverse and a direct relationship with frequency ,depending on the type of media .

Neither of the two is correct because light does not change frequency when it travels through different media .

Disclaimer :The question given on the portal is incomplete .Here is the complete question .

Question :Frank and Lisa are analyzing the chart, which shows the speed at which light travels through different media. Frank says that light travels through diamond with the greatest frequency. Lisa says that light travels through air with the greatest frequency. Which best describes the accuracy of these statements?

learn more from here: brainly.com/question/28098169

#SPJ4

A 2.50 × 105 w motor is used for 26.4 s to pull a boat straight toward shore. how far does the boat move toward shore if a force of 4.20 × 104 n is applied by the motor? 157 m 1,590 m 1.11 × 106 m 6.60 × 106 m

Answers

The distance travelled by the boat towards shore if a force of 4.20x10⁴N is applied is 157.14 m.

Power is a unit of measurement for the amount of work completed in a particular period of time; it may be computed using both work and time. Divide the amount of work by the length of time it takes to do the work to find power. The unit of work is the joule. The unit of power measurement is known as joules of work per second.

The power of the motor = 2.50x10⁵ W

                                     = 2.50x10⁵ N m/s

The time it takes to pull the boat = 26.4 s

The force applied  = 4.20x10⁴ N

Using the formula;

                                     [tex]d = \frac{Pt}{F}[/tex]

where,

P = Power

t = Time taken

F = Force applied

d = Distance travelled

The distance traveled by the boat = 2.50x10⁵ N m /s (26.4 s) /(4.20x10⁴ N)                                                

                                                      =157.14 m

Therefore, the distance travelled by the boat if a force of 4.20x10⁴N is applied is 157.14 m.

Learn more about power here:

https://brainly.com/question/4047067

#SPJ4

Which of the following is correct? Multiple Choice When AP is rising, MC is falling, and when AP is falling, MC is rising. When MP is rising, MC is falling, and when MP is falling, MC is rising. There is no relationship between MP and MC. When MP is rising, MC is rising, and when MP is falling, MC is falling.

Answers

The statement which is correct is that when MP is rising MC is falling, and when MP is falling MC is rising and is denoted as option B.

What is MP?

This is referred to as marginal product and is referred as the extra output derived as result of additional inputs.

MC on the other hand is known as marginal cost and they have an inverse relationship such that when MP is rising , MC is falling and vice versa thereby making it the most appropriate choice.

Read more about Marginal product here https://brainly.com/question/15314652

#SPJ1

an object has a mass of 50kg, a final height of 20m and an initial height of 8m. what is the amount of work done

Answers

amount of work done is 5880 J

Given:

mass of object = 50kg

Final height = 20m

initial height = 8m

To Find:

amount of work done

Solution:

work is done when a force acts upon an object to cause a displacement. You can calculate the energy transferred, or work done, by multiplying the force by the distance moved in the direction of the force.

The work done by gravity is given by the formula,

W = mgh

W = 50 x 9.8 x ( 20-8)

= 5880 J

So the work done is 5880 J

Learn more about Work done here:

https://brainly.com/question/25239010

#SPJ4

A mass m is attached to an ideal massless spring. When this system is set in motion, it has a period t. What is the period if the mass is doubled to 2m?.

Answers

The period is √2 *T if the mass is doubled to 2m.

What is the time period?

The time period is the length of time during which an activity occurs or a condition remains.

The equation for the period T is given as:

[tex]T = 2\pi \sqrt{\frac{m}{k} }[/tex]

where k is the spring constant

mass of the system = m

Here,

If the mass id doubled to 2m for the system that is set in motion,

new mass is now  2m

So the equation for time period becomes,

[tex]T' = 2\pi \sqrt{\frac{2m}{k} }[/tex]

[tex]T' = \sqrt{2} * 2\pi \sqrt{\frac{m}{k} }[/tex]

[tex]T' = \sqrt{2}* T[/tex]

Hence,

The period is √2 *T if the mass is doubled to 2m.

Learn more about the time period here:

https://brainly.com/question/17188760

#SPJ4

Two harmonic waves traveling in opposite directions interfere to produce a standing wave described by y

Answers

The wavelength of the interfering waves is 3.14 m.

Calculation:

The general equation of a standing wave is given by:

y = 2A sin (kx) cos (ωt)                            ......(1)

The given equation represents the standing wave produced by the interference of two harmonic waves:

y = 3 sin (2x) cos 5t                                .......(2)

Comparing equations (1) and (2):

k = 2

We know that,

k = 2π/λ

λ = 2π/k

λ = 2 (3.14)/ 2

λ = 3.14 m

Therefore, the wavelength of the interfering waves is 3.14 m.

I understand the question you are looking for is this:

Two harmonic waves traveling in opposite directions interfere to produce a standing wave described by y = 3 sin (2x) cos 5t where x is in m and t is in s. What is the wavelength of the interfering waves?

Learn more about interfering waves here:

https://brainly.com/question/2910205

#SPJ4

What mass will accelerate at 3 m/s² when a net force of 150 N acts on it?

Answers

Answer:

[tex]\huge\boxed{\sf m = 50\ kg}[/tex]

Explanation:

Given data:

Acceleration = a = 3 m/s²

Force = F = 150 N

Required:

Mass = m = ?

Formula:

F = ma

Solution:

Put the givens in the formula

150 = m (3)

Divide 3 to both sides

150/3 = m

50 kg = m

m = 50 kg

[tex]\rule[225]{225}{2}[/tex]

1. Determine the energy released per kilogram of fuel used.
Given MeV per reaction, calculate energy in joules per kilogram of reactants.
Consider 1 mole of plutonium-239 to be a mole of “reactions” (molar mass = 239 grams).

2. Determine the mass of fuel required for the expected energy consumption in the United States for the next 10 years.
Energy use per person per year in the United States = 3.5 × 1011 joules.
Base your calculations on a current population of 310,000,000.

Answers

The formula for energy release per kilogram of fuel burned is energy release per kg=6.702*10-13. and 19. J 1 Mev = 1.602 X 10 T

Calculate the energy in joules per kilogram of reactants given MeV per reaction. Energy is the ability or capacity to perform tasks, such as the ability to move an item (of a certain mass) by exerting force. Energy can exist in many different forms, including electrical, mechanical, chemical, thermal, or nuclear, and it can change its form.
Think of a mole of plutonium-239 (molar mass: 239 grams) as a mole of "reactions."

Energy used in the US per person annually = 3-5 X 1011
Population (number of people) = 3.108The required mass of the fuel is 3.5x1011 x3-1x10 8x 10)/6.703 X1013 kg. the mass required: 1.62 x 1033 kg Mev in Joules 6 is equal to 101.60*I0-
19. J 1 Mev = 1.602 X 10 T, which translates to 1.602*1013/2.39x10-3 energy release per kilogram, or 6.702*10-13.

To learn more about Energy please visit -
https://brainly.com/question/27671072
#SPJ1

Answer the following regarding Momentum Change and Force

1. Find the force necessary to gain a velocity of 60 m s-1 within 5 minutes, when acted on a mass of 200 kg which is initially at rest.

2. A force of 150 N acts on an object of mass 600 kg for 2 minutes, which was initially at rest. Find the velocity gained by the object at the end of the second minute.

Answers

The force necessary to gain the velocity is 40 N.

The velocity gained by the object at the end of the second minute is 30 m/s.

Force of the object

F = mv/t

where;

m is mass of the object = 200 kgv is velocity of the object = 60 m/st is time of motion = 5 mins = 300 seconds

F = (200 x 60)/(300)

F = 40 N

Velocity of the object

F = mv/t

Ft = mv

v = Ft/m

where;

m is mass = 600 kgF is force = 150 Nt is time = 2 mins = 120 s

v = (150 x 120)/(600)

v = 30 m/s

Thus, the force necessary to gain the velocity is 40 N.

The velocity gained by the object at the end of the second minute is 30 m/s.

Learn more about force here: https://brainly.com/question/12970081

#SPJ1

A new ride being built at an amusement park includes a vertical drop of 126.5 meters. Starting from rest, the ride vertically drops that distance before the track curves forward. If the velocity at the bottom of the drop is 10.0 m/s and the mass of the cart and passengers is 3.5 x 104 kg, how much potential energy was converted into thermal energy (heat)

Answers

The energies which is converted into thermal energy  is

2280.9 ×  10^4 J

According to the question when the ride will go upward then there will a potential energy acquired and when it comes down then kinetic energy starts to increase. since, the ride is starting from rest then kinetic energy will be maximum at lowest. So, now we will calculate P.E and K.E at various points.

Potential Energy at top when the vertical drop is 126.5 m is given as- P.E = m ×g× h

= 3.5 × 10^4 × 9.8 × 126.5

= 4338.95 × 10^4 J

Kinetic Energy at bottom when ride has velocity of 10 m/s then it is

given as -

K.E = 1/2 mv²

= 1/2 × 3.5×10^4 × 10²

= 175 × 10^4 J

Now the energy which converted to thermal energy is given as:

Thermal energy =  Potential energy - kinetic energy

= 2455.9 × 10^4 - 175 × 10^4

= 2280.9 × 10^4 J

Thus, the ride Potential energy was converted to thermal energy is 2280.9 × 10^4 J.  

Learn more about energies

here:https://brainly.com/question/12865479

#SPJ4

During World War II, the United States government forced thousands of _____ to be relocated to internment camps.

Answers

Answer:

Japanese Americans

Explanation:

prevented them from buying land and sent them back to japan after the war

hope this helps :)

Answer:

Japanese Americans

Explanation:

Since Japan was helping the Axis, the United States were afraid that if Japan attacked the Continental United States, then Japanese Americans would help them.

This fear led to the government placing Japanese Americans in internment camps.

A loop rests in the plane of a page of textbook while a magnetic field is directed into the page. A clockwise current is induced.

Answers

(b) when the magnetic field gets stronger.

correct option :

When a loop rest in the plane of the page and magnetic field is directed into the page then we will apply here right hand thumb rule in which thumb represent the direction of magnetic field and curling fingers represent direction of induced current. So, in this case the direction of magnetic field is into the page and curling fingers shows induced current in clockwise direction.

  Thus, option (b) is correct.

Incorrect options:

   option (A) is incorrect because when magnetic field will change  

     according to flux.

   option (C) is incorrect because field is stronger so it does not matter

     size of loop increases or decreases.

   option (D) is incorrect because it moves any other side the field will be

     stronger.

Learn more about magnetic field induction here:

 https://brainly.com/question/13610297

   #SPJ4

[Your question is incomplete, but most probably your full question was-Check all that apply:

A. when the magnetic field is tilted so it is no longer perpendicular to the page

B. when the magnetic field gets stronger

C. when the size of the loop decreases

D. when the loop is moved sideways across the page]

To lower the risk of a collision, you should keep at least __________ of space to one side of your vehicle at all times.

Answers

One must keep one car width of space to side of vehicle to avoid collision.

What is collision?

Collision is the sudden striking of two bodies that exerts forces on each other thus acelerating the objects in relatively very short time. One must maintain atleast one car width of space to side of vehicle to lower the risk of collision. If the space is lower than one car width than you should lower the speed of car to avoid collision.

One can also follow the two second disciple to avoid collision. According to this rule, the car should maintain two second safe distance behind any other vehicle .It acts as a safety buffer or defensive mechanism to avoid the risk of collision at all times at any speed. Also according to impulse momentum change theorem, the force on each object can be minimized by maintaing  more distance. As force and time are inversely proportional in collision.

Therefore, one car width distance is the safest minimum distance to avoid risks of collision .

Learn more about collision here:

https://brainly.com/question/14439547

#SPJ1

Cindy runs 2 kilometers every morning. She takes 2 minutes for the first 250 meters, 4 minutes for the next 1,000 meters, 1 minute for the next 350 meters, and 3 minutes for the rest.
Cindy’s average speed for the entire run is
meters per minute. One kilometer is the same as 1,000 meters.

Answers

Answer:

200 m / min

Explanation:

Total distance = 2000 m

Total time = 2+4+1+3 = 10 minutes

Average speed = 2000 m / 10 min = 200 m/min

What is the angular momentum of the moon around the earth? the moon’s mass is 7.4 * 1022 kg and it orbits 3.8 * 108 m from the earth.

Answers

The angular momentum of the moon around the earth is 2.92 * [tex]10^{34}[/tex] kg [tex]m^{2}[/tex]/s

Angular momentum a vector quantity that is a measure of the rotational momentum of a rotating body or system, that is equal in classical physics to the product of the angular velocity of the body or system and its moment of inertia with respect to the rotation axis, and that is directed along the rotation axis.

m (mass of moon ) = 7.4 * [tex]10^{22}[/tex] kg

orbit = 3.8 * [tex]10^{8}[/tex] m

L = I ω

  = m [tex]r^{2}[/tex]ω

  = 7.4 * [tex]10^{22}[/tex]  * [tex](3.8 *10^{8} )^{2}[/tex] * (2π / 27 * 24 * 3600)

  = 2.92 * [tex]10^{34}[/tex] kg [tex]m^{2}[/tex]/s

To learn more about angular momentum here

https://brainly.com/question/15104254?referrer=searchResults

#SPJ4

The path of a charged particle moving parallel to a uniform magnetic field will be a:______.

Answers

The path of a charged particle moving parallel to a uniform magnetic field will be straight line

Force on a charged particle moving in a magnetic field can be calculated by = q.(v*B )

q = charge of the particle

v = velocity of the particle

B = magnetic field

theta = angle between v and B

q . (v B sin (theta) )

Since,  particle is parallel to the magnetic field

hence , theta = 0°

so sin(theta) = sin 0° = 0

hence , force = 0

since , there is no force acting on the particle it will remain in that motion in hat it was when it initially came in the magnetic field . Hence It will be moving along a straight line path because the magnetic force on the charged particle is zero.

To learn more about magnetic field here

https://brainly.com/question/17011493

#SPJ4

A 75.0-kg man standing on a bathroom scale in an elevator. Calculate the scale in N, reading if the elevator moves upward at a constant speed of 1.5 m/s

Answers

A 75.0-kg man standing on a bathroom scale in an elevator, if the elevator moves upward at a constant speed of 1.5 m/s then the scale in N will be 885N accordance with Newton's law

 

The extent of the force being applied downward. If the human is selected to be the system of interest with a free-body diagram, the one-dimensional problem appears to be considerably more difficult. Both sections (a) and (b) of this example's analysis can be answered by analyzing the free-body diagram in accordance with Newton's laws, along with some other potential problems. In order to determine what the scale reads, the only forces operating on the subject are his weight w and the upward force of the scale, which are equal in strength and directed in opposing directions. Newton's law, as usual, can be used to do this.

F=Ma  = F−w=ma⇒F=m(a+g)=75(2+9.8)=885 N​.


To learn more about such problems on Newton's law please visit -
https://brainly.com/question/8052658
#SPJ4

How does enormous energy get released from the sun?

Answers

Enormous energy get released from the sun due to fusion reaction.

How does enormous energy get released from the sun?

Sun can generate enormous energy due to nuclear fusion. In this process, high temperature and pressure in the core of the sun force the nuclei to separate from their electrons which results in the  formation of one Helium atom releasing a large amount of energy.

So we can conclude that enormous energy get released from the sun due to fusion reaction.

Learn more about energy here: https://brainly.com/question/13881533

#SPJ1

Select the correct answers. which two pieces of information show that a research source is reliable?\

Answers

Answer:

2

Explanation:

just took the assignment

An ideal ammeter would have zero resistance while an ideal voltmeter would have infinite resistance, why?.

Answers

Answer: An ideal ammeter would have zero resistance, because to ensure that, there is no voltage drop due to the internal resistance. Similarly, an ideal voltmeter would have infinite resistance, because to ensure that there is no current is drawn by the voltmeter.

Explanation: To find the answer, we need to know about the Ammeter and Voltmeter.

What is an ammeter?An ammeter is a device, that can be used to measure the electric current flows through a circuit in amperes.An ideal ammeter would have zero resistance, because to ensure that, there is no voltage drop due to the internal resistance when it is connected in series to measure the current.What is voltmeter?A voltmeter is a device, that can be used to measure the electric potential difference generated between the terminals of an electric circuit in volts.An ideal voltmeter would have infinite resistance, because to ensure that there is no current is drawn by the voltmeter, when it is connected in parallel to measure the voltage.

Thus, we can conclude that, an ideal ammeter would have zero resistance, because to ensure that, there is no voltage drop due to the internal resistance. Similarly, an ideal voltmeter would have infinite resistance, because to ensure that there is no current is drawn by the voltmeter.

Learn more about the ammeter and voltmeter here:

https://brainly.com/question/28044897

#SPJ4

A transformer usefully transfers 190,000 W of the 200,000 W of energy supplied to it. How efficient is the transformer?

Answers

Energy efficiency = useful energy output/ total energy input
Therefore (190000/200000) x 100 (to work out percentage) = 95%

What is kinetic energy? A. stored energy B. the energy of motion C. the energy of light D. the energy at the top of a roller coaster​

Answers

Answer:

B. the energy of motion

Explanation:

Energy of motion. Kinetic energy is essentially the release of potential energy which is when an object has the potential to do work, due to it's position or molecular composition.

The hypothetical upper limit to the mass a star can be before it self-destructs due to the massive amount of fusion it would produce. Stars over this limit tend to be surrounded by massive nebulae of material blasted off of their surfaces.

Answers

The hypothetical upper limit to the mass a star can be before it self-destructs due to the massive amount of fusion it would produce is apparently as a result of Eddington luminosity

What are stars?

Stars are a fixed luminous point in the sky which is a large and remote incandescent body

So therefore, the hypothetical upper limit to the mass a star can be before it self-destructs due to the massive amount of fusion it would produce is apparently as a result of Eddington luminosity

Learn more about stars:

https://brainly.com/question/13018254

#SPJ1

Other Questions
Which branch of government was designed to make the federal government capable of timely and decisive action to deal with public issues and problems? What did mr esmond blame for many of his problems as a parent? Pretend that you are at the park babysitting. There are dozens of children playing on the playground. Use 3 onomatopoeias to describe the scene. Find the expected value of the winningsfrom a game that has the followingpayout probability distribution:Payout ($)1 2 5 8 10Probability 0.35 0.2 0.1 0.2 0.15Expected Value = [? ]Round to the nearest hundredth.Enter Which observation is not evdience that helped identify sodium a s the early current of the action potentioa?a. The current declined when there was decreased driving force on sodium fluxes.b. The current disappeared near the Nernst potential for sodium.c. The early current was blocked by tetrodotoxin.d. The early current was unaffected by tetraethylammonium.e. When the late current was blocked, the reversal potential of the inward current shifted to a negative membrane potential. Clark works for Acme Bank and asks for your assistance. He wants you to look at a property that's about to go into foreclosure and provide him with a report on its current marketability and an approximate market value. What's Clark most likely asking you to provide The ill young woman was pushed from the sky because creative response1. learner write their own folklore( any types of their own)2. a learner may present a transactional writing like a poster or comic trip A see-saw is balanced on itsmidpoint. A 22.4 kg kid sits on the left, 1.75 m from the axis. A second kid sits on the other side, 1.22 m from the axis. What is the mass of the second kid? 11 points please help me choose the correct option that explains what steps were followed to obtain the system of equations below x+6y = 5 According to the Atalay et al 2017 comic book reading, how should Historical Societies and Private Collectors handle human remains What is the irs underpayment penalty for an overstated charitable contribution deduction taken directly on form 1040?. please help with this question Forrest Gump, the latest cinematic impersonation of American innocence, has become an icon dear to the hearts of the baby-boom generation whose common experience he mirrors. Gumps life, it seems, is how millions of middle-aged Americans feel theirs should have been (Charles Laurence, Daily Telegraph)1. What does Charles mean when he says this audience shares a common experience with the character of Forrest Gump? Explain. 2. Do you feel that without this common experience ones enjoyment of the film Forrest Gump might be lessened? Explain. 20pts and brainliest !! please help quick What does citizenship in the nation mean? Quick algebra 1 assignment for 50 points!Only answer if you know the answer, tysm!1. Create 5 questions referencing Relations and Functions Below is an example of this.Example : Is the set of ordered pairs below a function? Yes or no, explain your reasoning. {(4,6),(10,-2),(13,7),(15,8),(10,11)}2. Answer each question and show a brief step by step process on how you got the answer to each of your questions. When the total pressure inside a tennis ball reaches 1013 kPa, the tennis ball will burst. If the balloriginally has a pressure of 405 kPa at room temperature (20C), at what temperature will the ball burst?(submit your work) Josue has a coin collection. He keeps 11 of the coins in his box, which is 5% of the collection. How many total coins are in his collection?